Last visit was: 19 Nov 2025, 05:45 It is currently 19 Nov 2025, 05:45
Close
GMAT Club Daily Prep
Thank you for using the timer - this advanced tool can estimate your performance and suggest more practice questions. We have subscribed you to Daily Prep Questions via email.

Customized
for You

we will pick new questions that match your level based on your Timer History

Track
Your Progress

every week, we’ll send you an estimated GMAT score based on your performance

Practice
Pays

we will pick new questions that match your level based on your Timer History
Not interested in getting valuable practice questions and articles delivered to your email? No problem, unsubscribe here.
Close
Request Expert Reply
Confirm Cancel
User avatar
Sajjad1994
User avatar
GRE Forum Moderator
Joined: 02 Nov 2016
Last visit: 18 Nov 2025
Posts: 17,289
Own Kudos:
49,302
 [8]
Given Kudos: 6,179
GPA: 3.62
Products:
Posts: 17,289
Kudos: 49,302
 [8]
2
Kudos
Add Kudos
6
Bookmarks
Bookmark this Post
avatar
biancanancy
Joined: 05 Feb 2022
Last visit: 11 Jun 2022
Posts: 4
Own Kudos:
Given Kudos: 6
Posts: 4
Kudos: 1
Kudos
Add Kudos
Bookmarks
Bookmark this Post
User avatar
Sajjad1994
User avatar
GRE Forum Moderator
Joined: 02 Nov 2016
Last visit: 18 Nov 2025
Posts: 17,289
Own Kudos:
Given Kudos: 6,179
GPA: 3.62
Products:
Posts: 17,289
Kudos: 49,302
Kudos
Add Kudos
Bookmarks
Bookmark this Post
User avatar
thakurarun85
Joined: 10 Jul 2021
Last visit: 21 Sep 2022
Posts: 224
Own Kudos:
Given Kudos: 29
Posts: 224
Kudos: 53
Kudos
Add Kudos
Bookmarks
Bookmark this Post
Sajjad1994

please provide solution to question 4.
User avatar
Sajjad1994
User avatar
GRE Forum Moderator
Joined: 02 Nov 2016
Last visit: 18 Nov 2025
Posts: 17,289
Own Kudos:
Given Kudos: 6,179
GPA: 3.62
Products:
Posts: 17,289
Kudos: 49,302
Kudos
Add Kudos
Bookmarks
Bookmark this Post
thakurarun85
Sajjad1994

please provide solution to question 4.

Official Explanation

4. Achieving an agreement at the latest round of trade talks seems doubtful because

Explanation

Paragraphs 1 and 3 illustrate that both some EU countries and the G20 group of developing nations are unsure of the costs and gains resulting from a new trade agreement. Note that (C) is just the opposite of what is contained in the passage, at the end of the first paragraph.

Answer: D
User avatar
Jarvis07
Joined: 06 Sep 2017
Last visit: 19 Nov 2025
Posts: 295
Own Kudos:
236
 [1]
Given Kudos: 160
GMAT 1: 750 Q50 V41
GMAT 1: 750 Q50 V41
Posts: 295
Kudos: 236
 [1]
1
Kudos
Add Kudos
Bookmarks
Bookmark this Post
Quote:
1. It can be inferred that a removal of agricultural distortions would provide gains to poor countries of approximately

(A) $300 million per year.
(B) $500 million per year.
(C) $100 billion per year.
(D) $300 billion per year.
(E) $500 billion per year.


I love when I get an opportunity to do maths in Verbal questions. So here we go again:

This question comes from following part of the passage:
The World Bank estimates that removing current agricultural distortions would produce a general benefit of more than $300 billion a year. Relative to national income, poor countries would enjoy a third more of this benefit than rich, industrialized ones.

If we let x be the amount that goes to developed countries, then poor countries would receive 4x/3 (since they would receive one-third more than developed countries).

So the total gain from removing agricultural distortions would be x + 4x/3 = 7x/3. We know that this total gain is estimated to be more than $300 billion per year. Therefore, we can set up the equation:

7x/3 = $300 billion

Solving for x, we get:

x = ($300 billion x 3)/7 = $128.6 billion or Approximately $100 billion
Hence C.
User avatar
Jarvis07
Joined: 06 Sep 2017
Last visit: 19 Nov 2025
Posts: 295
Own Kudos:
236
 [1]
Given Kudos: 160
GMAT 1: 750 Q50 V41
GMAT 1: 750 Q50 V41
Posts: 295
Kudos: 236
 [1]
1
Kudos
Add Kudos
Bookmarks
Bookmark this Post
Quote:
2. The Indian government aims to

(A) penetrate Chinese consumer markets.
(B) reduce barriers to the sale of its farm produce.
(C) impose more restrictive trade policies.
(D) improve its bargaining position.
(E) protect its farmers against foreign competition.

The passage states that India is "refusing to negotiate without deeper concessions on agriculture" because its government worries that competition from Chinese factories and American farms represents too great a threat, while gaining more access to world markets is only of limited attraction. This implies that India aims to reduce barriers to the sale of its farm produce, as stated in answer choice (B).


A) Penetrate Chinese consumer markets: The passage does not mention anything about the Indian government trying to enter the Chinese consumer markets.

C) Impose more restrictive trade policies: The passage does not suggest that the Indian government aims to implement more restrictive trade policies.

D) Improve its bargaining position: Although the Indian government is looking for deeper concessions on agriculture, there is no direct indication that this is being done to improve its bargaining position.

E) Protect its farmers against foreign competition: This is somewhat related to the correct answer of reducing barriers to the sale of farm produce, but it is not explicitly stated in the passage that the Indian government is looking to protect their farmers.
User avatar
Jarvis07
Joined: 06 Sep 2017
Last visit: 19 Nov 2025
Posts: 295
Own Kudos:
Given Kudos: 160
GMAT 1: 750 Q50 V41
GMAT 1: 750 Q50 V41
Posts: 295
Kudos: 236
Kudos
Add Kudos
Bookmarks
Bookmark this Post
Quote:
3. According to the passage, the American legislature is

(A) liberalizing its policy toward foreign trade.
(B) sensitive to the demands of third-world countries.
(C) becoming more protectionist.
(D) pushing hard for trade liberalization.
(E) opposing any amendment to existing world trade agreements.


The correct answer is (C) becoming more protectionist.


The passage mentions that the American politicians are "once again turning protectionist," and that the congress only barely passed the Bush administration’s Central America Free Trade Agreement, even though its impact on the American economy will be tiny compared to the ambitions for Doha.
User avatar
Jarvis07
Joined: 06 Sep 2017
Last visit: 19 Nov 2025
Posts: 295
Own Kudos:
236
 [1]
Given Kudos: 160
GMAT 1: 750 Q50 V41
GMAT 1: 750 Q50 V41
Posts: 295
Kudos: 236
 [1]
1
Kudos
Add Kudos
Bookmarks
Bookmark this Post
Quote:
4. Achieving an agreement at the latest round of trade talks seems doubtful because

(A) the EU trade commissioner has failed to persuade India and China to liberalize exports of farm produce.
(B) America’s fast-track negotiating authority is soon to expire.
(C) poorer countries refuse to lower agricultural barriers until developed countries open their markets to manufactured goods.
(D) both the EU and G20 groups are unsure of the gains that might accrue from an agreement.
(E) Britain and France are steadfastly against a new agreement.


Option (D) is the correct answer.

The passage states that both the EU and G20 groups are unsure of the gains that might accrue from an agreement, which suggests that they may not be fully committed to reaching an agreement.
Option (A) is incorrect because while the EU trade commissioner has indeed been trying to persuade India and China to liberalize exports of farm produce, this is not the main reason why achieving an agreement seems doubtful.
Option (B) is incorrect because America's fast-track negotiating authority expiring soon is not mentioned as a reason why achieving an agreement seems doubtful.
Option (C) is incorrect because while it is true that poorer countries have demanded that developed countries open their markets to manufactured goods before they lower agricultural barriers, this is not mentioned as the reason for the doubtfulness of achieving an agreement.
Option (E) is incorrect because there is no mention of Britain and France being against a new agreement in the passage.
User avatar
Susarla96
Joined: 24 Apr 2022
Last visit: 25 Nov 2024
Posts: 90
Own Kudos:
Given Kudos: 525
Concentration: Strategy, Operations
GMAT 1: 620 Q44 V31
GMAT 1: 620 Q44 V31
Posts: 90
Kudos: 118
Kudos
Add Kudos
Bookmarks
Bookmark this Post
Jarvis07
Quote:
3. According to the passage, the American legislature is

(A) liberalizing its policy toward foreign trade.
(B) sensitive to the demands of third-world countries.
(C) becoming more protectionist.
(D) pushing hard for trade liberalization.
(E) opposing any amendment to existing world trade agreements.


The correct answer is (C) becoming more protectionist.


The passage mentions that the American politicians are "once again turning protectionist," and that the congress only barely passed the Bush administration’s Central America Free Trade Agreement, even though its impact on the American economy will be tiny compared to the ambitions for Doha.

Hey, thanks for the explanation. Even though I thought that C is the right answer, I was unsure to select it because in the passage it says "American Politicians" and not "Legislature".

So please tell me if "politicians" in general means "legislature"? Or did I miss something?

TIA :)
avatar
adityakaregamba
Joined: 04 Mar 2024
Last visit: 30 Aug 2024
Posts: 22
Own Kudos:
Given Kudos: 457
GRE 1: Q165 V149
GRE 1: Q165 V149
Posts: 22
Kudos: 9
Kudos
Add Kudos
Bookmarks
Bookmark this Post
I'm still having trouble understanding how the answer to Q2 can be "reduce barriers to the sale of its farm produce." If India were to reduce barriers, that would mean other countries could export their products to India at a very competitive price. This could directly impact Indian markets, and ultimately lead to an increase in unemployment rates.

Can someone help me understand where I am going wrong ?
User avatar
manavsaraf001
Joined: 13 Jul 2022
Last visit: 14 Sep 2025
Posts: 15
Own Kudos:
Given Kudos: 240
Posts: 15
Kudos: 7
Kudos
Add Kudos
Bookmarks
Bookmark this Post
Jarvis07

Quote:
1. It can be inferred that a removal of agricultural distortions would provide gains to poor countries of approximately

(A) $300 million per year.
(B) $500 million per year.
(C) $100 billion per year.
(D) $300 billion per year.
(E) $500 billion per year.
I love when I get an opportunity to do maths in Verbal questions. So here we go again:

This question comes from following part of the passage:
The World Bank estimates that removing current agricultural distortions would produce a general benefit of more than $300 billion a year. Relative to national income, poor countries would enjoy a third more of this benefit than rich, industrialized ones.

If we let x be the amount that goes to developed countries, then poor countries would receive 4x/3 (since they would receive one-third more than developed countries).

So the total gain from removing agricultural distortions would be x + 4x/3 = 7x/3. We know that this total gain is estimated to be more than $300 billion per year. Therefore, we can set up the equation:

7x/3 = $300 billion

Solving for x, we get:

x = ($300 billion x 3)/7 = $128.6 billion or Approximately $100 billion
Hence C.
­As per your calculation , x is 128 billion . But the question is asking to find "gains to poor countries " , that should be 4x/3 .  Which comes out approx 42 billion . 

Please explain the solution again ­
User avatar
cosmic_matters1
Joined: 21 Sep 2023
Last visit: 19 May 2024
Posts: 8
Own Kudos:
Given Kudos: 23
Posts: 8
Kudos: 16
Kudos
Add Kudos
Bookmarks
Bookmark this Post
Susarla96

Jarvis07

Quote:
3. According to the passage, the American legislature is

(A) liberalizing its policy toward foreign trade.
(B) sensitive to the demands of third-world countries.
(C) becoming more protectionist.
(D) pushing hard for trade liberalization.
(E) opposing any amendment to existing world trade agreements.
The correct answer is (C) becoming more protectionist.

The passage mentions that the American politicians are "once again turning protectionist," and that the congress only barely passed the Bush administration’s Central America Free Trade Agreement, even though its impact on the American economy will be tiny compared to the ambitions for Doha.
Hey, thanks for the explanation. Even though I thought that C is the right answer, I was unsure to select it because in the passage it says "American Politicians" and not "Legislature".

So please tell me if "politicians" in general means "legislature"? Or did I miss something?

TIA :)
­The Vice President of the United States serves as President of the Senate and may cast the decisive vote in the event of a tie in the Senate.(Copied from the legislative branch of the US website). So as per my knowledge, The legislature comprises of politicians, that is why if majority of the politicians are protectionist,then the whole legislature is taken in to account.­
Moderators:
GMAT Club Verbal Expert
7443 posts
GMAT Club Verbal Expert
231 posts
GRE Forum Moderator
17289 posts
188 posts